Solve 7 - 2x = x - 14

Answers

Answer 1

Answer:

7 = x

Step-by-step explanation:

7 - 2x = x - 14

7 + 14 = x + 2x

21 = 3x

7 = x

Hope this helps!


Related Questions

WILL MARK BRAINLEST!!

Solve for y put in slope intercept form

(y = mx + b) to find slope
Y + 6 = 1/2 ( x + 20)

Show work please

Answers

First, begin by distributing 1/2 2X and 20. And then subtract six from each side. This is your answer. (Y=1/2x + 4)

*For slope intercept form, you literally just find the slope, and the y-intercept

Hope this helps bro!

Larry spends a total of 45 minutes running. He begins at a pace of 8 miles per hour and then reduces his speed to 5 miles per hour. Let t represent the number of minutes Larry spent running at the faster pace. Larry models his distance with the expression 8t+5(45−t) to model the situation. Did Larry correctly model the situation

Answers

Answer:

Yes, he did

Step-by-step explanation:

We have that:.

Total time = 45

At time t, he moves at 8mph.

This gives: 8t

The time left is: 45 - t and he moves at 5mph, this gives 5(45 - t).

Hence, the total is

8t + 5(45 - t)

By comparison, we can conclude that Larry modeled the situation correctly.

Use the Factor Theorem to determine whether h(x) = x – 1 is a factor of f(x) = 14x^99 – 65x^56 + 51.

a. Yes
b. No

Answers

It’s is yes hopes that helps lol
the answer is a (yes)

2 (k + 2) = 8k - 5 - 3k

Answers

Answer:

k=3

Step-by-step explanation:

Answer:k=3

Step-by-step explanation:

A scared tarantula was able to go a distance of 20 meters in 6 seconds. What was its
average speed?

Answers

Answer:

3.33 m/s

Step-by-step explanation:

speed = [tex]\frac{meters}{seconds} > \frac{20}{6} = 3.33[/tex]

PLEASE HELP FAST. WILL MARK AS BRAINLIEST IF GIVEN GOOD ANSWER ​

Answers

Answer:y=9

Step-by-step explanation:

What’s 5 divided by 6.2

Answers

5 divided by 6.2 is 0.80

Answer:

8

Step-by-step explanation:

Because you need to round 0.8064516129 to the nearest tenth

6 Ms. Flynn's class is painting a circular canvas. Its diameter is 5 feet. What is
the area of the canvas to the nearest whole number? Use 3.14 for .

Answers

19.625 ft
3.14(2.5 squared) =19.625

h(x)=x−11 i don't get it

Answers

If your are looking for the x it is x=11

which one does not belong ?
1/20 _20/25 _2/3_ 5/4
And why is the answer correct?.​

Answers

Answer:

2/3 because they have something to do with five

Step-by-step explanation:

What is the area of this figure?
13 in
6 in
8 in
s in
5 in
17 in
10 in
Write your answers using decimals, If necessary.

Answers

Answer:

If i am correct the area should 337inches..........

Step-by-step explanation:

my reason is becuase if you multiply these numbers

13* 6 = 78       and   8 * 8 = 64     and   10 * 17 + 170   and  last 5 * 5 = 25

they you add    170+ 78 + 64 + 25 =  337inches          so means the answer is 337 Your Welcome :D

Somebody help me plssss

Answers

Answer:

The last one 4x4

Step-by-step explanation:

Answer:

last one, D.

Step-by-step explanation:

A line passes through the points (7, 10) and (7, 20). Which statement is true about the line?
It has a slope of zero because x 2 minus x 1 in the formula m = StartFraction y 2 minus y 1 Over x 2 minus x 1 EndFraction is zero, and the numerator of a fraction cannot be zero.
It has a slope of zero because x 2 minus x 1 in the formula m = StartFraction y 2 minus y 1 Over x 2 minus x 1 EndFraction is zero, and the denominator of a fraction cannot be zero.
It has no slope because x 2 minus x 1 in the formula m = StartFraction y 2 minus y 1 Over x 2 minus x 1 EndFraction is zero, and the numerator of a fraction cannot be zero.
It has no slope because x 2 minus x 1 in the formula m = StartFraction y 2 minus y 1 Over x 2 minus x 1 EndFraction is zero, and the denominator of a fraction cannot be zero.

Answers

Answer: D) It has no slope because x 2 minus x 1 in the formula m = StartFraction y 2 minus y 1 Over x 2 minus x 1 EndFraction is zero, and the denominator of a fraction cannot be zero.

Step-by-step explanation:

slope is y2-y1/x2-x1

20-10/7-7

10/0

denominator can't be 0

Answer:

D

Step-by-step explanation:

cause i said so

which is bigger. 34 1/2 or 34.5

Answers

Answer: They are equal.

Step-by-step explanation:

34 1/2 equals 34.5 in decimal form

34.5 equals 34 1/2 in fractional form

(.5 equals 1/2)

What is the measure of 1?

Answers

Answer:

93

Step-by-step explanation:

A system of equations is shown below:

2x = 5y + 4
3x − 2y = −16

What is the solution to this system of equations? (5 points)

Answers

Answer:

x = -8, y = -4

Step-by-step explanation:

First we want to write the equations so the variables line up

[tex]2x - 5y = 4\\3x - 2y = -16\\[/tex]

Next, we could either multiply one of the equations to cancel out the x's or the y's, either one works.

I will choose to cancel out the x's

[tex]-3R_1 + 2R_2\\-6x + 15y = -12\\6x - 4y = -32[/tex]

add the equations and solve

[tex]11y = -44\\y = -4[/tex]

once you have one variable, you can use it to solve the other

[tex]2x - 5(-4) = 4\\2x + 20 = 4\\2x = -16\\x = -8[/tex]

verify

[tex]3(-8) - 2(-4)\\-24+ 8\\-16\\[/tex]

it works

So the answer is
X = -8
Y = -4
Here is the working

what type of number is -15

Answers

The answer is: Negative number

Answer:

an integer

Step-by-step explanation:

integers are whole number that includes positive number, negative numbers, and zero

Write the inverse function for the function, ƒ(x) =x + 4. Then, find the value of ƒ -1(4). Type your answers in the box.

ƒ -1(x) =

ƒ -1(4) =

Answers

Answer:

f-1(x)=x-4

f-1(4)=0

Step-by-step explanation:

to get an inverse function, replace x with y, and y with x.

y=x+4  ---> x=y+4

solve for y: x=y+4 --> y=x-4 --> f-1(x)=x-4

f-1(4)=(4)-4=0

4.5x-1.5y=5.4 solve equation for y

Answers

Answer:

y=-3.6+3x

Step-by-step explanation:

What is the value of x x+18+5x-6=180

Answers

Step-by-step explanation:

I Hope it helps you . Hope it will

Answer:X=28

Step-by-step explanation:

first you combine the like terms wich are x and 5x also -6 and 18

second your equation will be 6x+12=180

Third you subtract 12 from each side

4th you divide each side by 6 to simplify the x

5th your anwse would be x =28

Please answer will mark brainliest

Answers

Okkkkkkkkkk
But what’s the question I know I was second but can I have is I’m trynna get 500 points so I can text people

HELPPP !! LOOK AT PICTURE
The function, f(x), is a third-degree polynomial whose
zeros are shown on the number line. Use the function
values below to determine the solutions set of f(x) < 0.

Answers

Answer:

A) (-infinity,-2] and D) [0,2]

Step-by-step explanation:

At a quiz competition for every question team A answered correctly team B answered 2 questions correctly. The ratio of the number of questions team A answered correctly to the number of questions team B answered correctly is ___ 1 to1 2 to 1 1to2 2to2

Answers

Answer:

1 to 2

Step-by-step explanation:

If A answers 1 question, B will answer 2 questions, making the ratio 1 to 2

Answer:

The answer is 1 to 2 i did the quiz on flvs

Step-by-step explanation:

A local carpenter uses the function LaTeX: f\left(x\right)=180xf ( x ) = 180 x to determine the amount earned after selling LaTeX: xx number of book cases. He has to sell between 5 and 10 bookcases each month in order to meet his sales goal. Find the domain for this situation. Group of answer choices

Answers

Given:

A local carpenter uses the function f(x) to determine the amount earned after selling x number of book cases.

[tex]f(x)=180x[/tex]

He has to sell between 5 and 10 bookcases each month in order to meet his sales goal.

To find:

The domain for this situation.  

Solution:

We have,

[tex]f(x)=180x[/tex]

where, f(x) is the amount earned be carpenter after selling x number of book cases.

Domain is the set of input values.

Here, input values are number of books he has to sell, which can be a fraction value.

Since, he has to sell between 5 and 10 bookcases each month in order to meet his sales goal, therefore the x values must be integers between 5 and 10.

Therefore, the domain for this situation is set of integers between 5 and 10, i.e., {5,6,7,8,9,10}.

what is the equation in slope intercept form 6x-4y=12 ?​

Answers

Answer:

y=6/4x-3

Step-by-step explanation:

First, you isolate the y, by subtracting 6x from each side. Then, you divide by -4 on both sides to isolate it to an individual y.

Answer:

Slope = 6/4 or 1 1/2

Step-by-step explanation:

Step 1:

6x - 4y = 12       Equation

Step 2:

y = mx + b       Slope Intercept Form

Step 3:

- 4y = - 6x + 12      Subtract 6x on both sides

Step 4:

y = 6/4x + 12        Divide 4 on both sides

Answer:

Slope = 6/4 or 1 1/2

Hope This Helps :)

Alright then! First question!
A graph has height (inches) on the x-axis and arm span (inches) on the y-axis. Points are at (30, 27), (45, 45), (53, 50), (60, 60), (67, 70), and (73, 72).

Identify if there is a relationship between the variables.
(A) No, there is no relationship because the points are all up and down.
(B) Yes, the relationship displayed shows arm span increasing as height increases.
(C) Yes, the relationship displayed shows arm span decreasing as height increases.

Answers

Answer:

hmmm...i wanna go with B

Step-by-step explanation:

because it goes from 30 to 45 and 45 and 45 to 53 to 50 then the 50 goes 60 meaning their both increasing as it it goes on

at least i believe im right haha hope this helped if its not to  late

Answer: The answer is (B) Yes, the relationship displayed shows arm span increasing as height increases.

Step-by-step explanation: I got the answer correct hope this helps!

Erica and 4 friends are going out for pizza for dinner. They split one pizza and 5 small drinks. The pizza costs $13.00. They spend a total of $20.50. Find the cost of one small drink. Define the variable, write and solve an equation, and answer the question.

Answers

to find the cost of a small drink, you might want to:

subtract the decimals

total cost : $20.50

cost of pizza :$13.00.

cost of one small drink : total cost - cost of pizza

                                      :  $20.50 - $13.00.

                                      : 20.50

                                    -   13.00

                                      :    07. 50

                                     :   $ 7.50

                                 

What is the range of the following graph?

Answers

Answer: 6

Step-by-step explanation: 8 - 2 = (6) Range is the difference between the highest and lowest number just for future references. Hope this helped! :)

Answer: i think is 8 also that person already explain

Step-by-step explanation: have a nice day

Ken went to a county fair and spent $5 on admission, $6.50 on games, and $7.21 on food. If he had $30 before he went to the fair, how much money does he have left?

Answers

Answer:

11.29 left

Step-by-step explanation:

30 - 5 = 25 - 6.50 = 18.50 - 7.21= 11.29

Answer:

He would have $11. 29 left

Step-by-step explanation:

30-5 =25

25. 00-6.50 = 18.50

18. 50-7.21 = 11.29

Brainliest to the correct answer ( Evaluate 7 - 5p + 3q when p = 1 and a = 7.)

Answers

Answer:

23

Step-by-step explanation:

7 - 5p + 3a

7 - 5(1) + 3(7)

7 - 5 + 21

2 + 21

23

the answer would be 23

7-5p+3a
7-5(1)+3(7)
7-5+21
2+21
23

Plug in your numbers !!
Other Questions
Help!!Which of the following describes the first and foremost guideline in dimensioning? A. Legibility B. Measurement C. Accuracy D. Sketching Multiply. 2(4)(3)(5) 120 4 4 120 The phrase "Renaissance Man" has been used to describe Leonardo da Vinci. Why is this phrase used to describe Leonardo da Vinci? Solve: 3(g 2) = 8 Is burning the glass plate is physical or chemical change? Please ANSWER HURRY?!What did de Niza say he saw at Hawakuh? Why is water scarcity a problem in the 21st century? (Site 1) What is the solution to the linear equation?12 + 3b 1 = 5 bb = 2b = 1.5b = 1.5b = 2 Find the measure of an angle such that the sum of its supplement and four times itscomplement is 13 less than twice the angle.Show work Whats the highest common multiple? Does anyone know what instant gratification is? Who must approve all appointments of federal judges?A presidentB SenateC House of RepresentativesD Supreme Court How are lines KL and MN related?O The lines intersect at point K.O The lines are parallel.O The lines are perpendicular.O The lines do not have slopes, help me pleaseeeeeeeeeeeeeeeeeeeeeee Suppose there is a 1.7f drop in temperature for every thousand feet that an airplane climbs into the sky. If the temperature on the ground is 49.9f what will be the temperature when the plane reaches am altitude of 4,000ft?? The temperature will be_____F (type an integer or a decimal) did I get these correct? 2) If 20 workers can build a house in 45 days, how many more workers are neededto complete it in 30 days?6B1510D18 Which expression is equal to 527.471?*(5x100)+(2x10)+(7x1)+(4x10) + (7x100)+ (1x1,000)(5x100,000)+(2x10,000)+(7x1,000)+(4x100) + (7x10)+ (1x1)(5x100)+(2x10)+(7x1)+(4x1/1) + (7x1/10)+ (1x1/100)(5x100)+(2x10)+(7x1)+(4x1/10) + (7x1/100)+ (1x1/1,000) which algebraic expression represents the phrase "the quotient of negative eight and the sum of a number and three" What is the hypothesis in this conditional statement?If it is the holiday season, then the post office is crowded. 1. It is the holiday season 2. The post office is crowded